What is Limits: Definition and 1000 Discussions

Els Límits (Catalan pronunciation: [əlz ˈlimits]) is a Spanish village, a civil parish of the municipality of La Jonquera, situated in the province of Girona, Catalonia, in Spain. As of 2005 its population was of 115. Its Spanish name is Los Límites.

View More On Wikipedia.org
  1. Nipuna Weerasekara

    What is the limit of lnx as x approaches a negative number?

    Homework Statement Find the following limit.Homework Equations The Attempt at a Solution I cannot apply L' Hopital rule because it does not apply to this question. Hence I have no idea how to approach to this question. Please give me some guidelines.
  2. Nemo1

    MHB Solving Limits and Riemann Sums: Tips from Nemo

    Hi Community, I have the following question: I have done basic solving of limits and also of Riemann sums but never had to do them in the same question. Would I be correct in saying that I need to solve for the Riemann sum first then take the limit of the integral? Cheers Nemo
  3. U

    Limits and Derivatives: Solving lim[2sin(x-1)/(x-1)] as x approaches 1

    Homework Statement What will be lim[2sin(x-1)/(x-1)], where x tends to 1? [ ] denotes greatest integer function. Homework Equations Can I directly solve it using the formula sinx/x =1 when x tends to 0 The Attempt at a Solution Okay so the quantity inside [ ] can be written as ——>>2...
  4. Velo

    MHB Two Similar Limits with Different Results

    So, I'm still struggling with limits a bit.. Today, I've tried solving two different exercises which look pretty much the same. I could solve the first one relatively easily: \lim_{{x}\to{+\infty}}\frac{\sqrt{4x^{2}-1}-x}{x-3} I applied the usual steps and arrived to the expression...
  5. BreCheese

    Using series to prove hypothesis of right triangle's < limits

    Homework Statement I need to mathematically prove that the center angle(s) (labeled as "A" in the photo below) approach what I believe to be 60 degrees (but never reach 60 degrees). We are given the values of all longer legs of each right triangle. Furthermore, the value of the length of each...
  6. Y

    MHB Calculating Limits without Cheating or L'Hopital's Rule

    Hello all, I am trying to calculate the following limits, without cheating and using a calculator (by setting a very close value of the required value of x). And no l'hopital's rule either if possible :-) The limits are: \[\lim_{x\rightarrow 0} \frac{ln(x^{2}+e^{x})}{ln(x^{4}+e^{2x})}\]...
  7. JulienB

    I General questions about limits of sequences

    Hi everybody! I'm currently preparing a math exam, and I'd like to clear up a few points I find obscure about limits of sequences, my goal being to more or less determine a method to solve them quickly during the exam. Hopefully someone can help me here :) I'll number the questions so that it's...
  8. Jess Karakov

    Sequence Convergence/Divergence Question

    Homework Statement Determine which of the sequences converge or diverge. Find the limit of the convergent sequences. 1) {asubn}= [((n^2) + (-1)^n)] / [(4n^2)] Homework Equations [/B] a1=first term, a2=second term...an= nth term The Attempt at a Solution a) So I found the first couple of...
  9. W

    Surface Integral Limits: Solving for u and v

    Homework Statement Problem is in image uploaded Homework Equations n/a The Attempt at a Solution x = u, y = v and z = 1 - u - v ∂r/∂u × ∂r/∂v = i + j + k F dot N = u^2 + 3v^2 ∫∫(u^2 + 3v^2 )dudv My problem is I'm not sure what I should take as the limits? Should I flip around the order of...
  10. thegirl

    I How Does the Limit of cosh and sinh Approach 1?

    Hi I was wondering how you get this when taking the limit of T going to 0 From this expression of S: Please help I don't see how ln infinity goes to uB/KbT (used u to represent the greek letter. And how does the other expression of sinh and cosh approach 1?
  11. Ethan Godden

    Is the Series Convergent or Divergent?

    Homework Statement I am supposed to determine whether the summation attached is convergent or divergent Homework Equations Alternating Series Test Test for Divergence The Attempt at a Solution The attempted solution is attached. Using the two different tests I am getting two different answers.
  12. G

    Limit of arccosh x - ln x as x -> infinity

    Homework Statement find the limit of arccoshx - ln x as x -> infinity Homework Equations ##arccosh x = \ln (x +\sqrt[]{x^2-1} )## The Attempt at a Solution ## \lim_{x \to \infty }(\ln (x + \sqrt{x^2-1} ) - \ln (x)) = \lim_{x \to \infty} \ln (\frac{x+\sqrt{x^2-1}}{x}) \ln (1 + \lim_{x \to...
  13. gsmtiger18

    Limits of sequences involving factorial statements

    Homework Statement I have to determine whether or not the following sequence is convergent, and if it is convergent, I have to find the limit. an = (-2)n / (n!) In solving this problem, I am not allowed to use any form or variation of the Ratio Test. 2. The attempt at a solution I was...
  14. S

    Exploring the Limits of f(x,y): Why (x,0) and (0,y) Matter

    Homework Statement When we are taking a limit of a multivariable function, why do we use the points (x,0) and (0,y) to find out if the limit doesn't exist? They are two different points, no? If they are two different points then wouldn't they go to different points on the z-axis? If we have a...
  15. Euler2718

    Help with Multi-variable Limits

    Homework Statement Evaluate or show that the limit does not exist: \lim_{(x,y) \to (0,0)}\frac{ 2x^{4} + 5y^{3} }{8x^{2}-9y^{3}} \lim_{(x,y) \to (0,-2)}\frac{ xy+2x }{3x^{2}+(y+2)^{2}} Homework EquationsThe Attempt at a Solution So the first one is indeterminate and cannot be factored...
  16. evinda

    MHB What does this Limit mean geometrically?

    Hello! (Wave) I want to find the following limit, if it exists. $\lim_{(x,y) \to (0,0)} \frac{\cos x-1-\frac{x^2}{2}}{x^4+y^4}$ If we say : let $(x,y) \to (0,0)$ along the line $y=0$ , what exactly does it mean geometrically? Also, if we want to check whether the limit $\lim_{(x,y) \to...
  17. F

    A Taking limits in discrete form

    I'm trying to calculate this limit to answer a question in Quantum Mechanics: \mathop {\lim }\limits_{{t_1} \to 0} \,\,{\left( {\frac{m}{{2\pi \hbar i{t_1}}}} \right)^{{\raise0.5ex\hbox{$\scriptstyle 1$} \kern-0.1em/\kern-0.15em \lower0.25ex\hbox{$\scriptstyle 2$}}}}{e^{im{{(x' - x)}^2}/2\hbar...
  18. C

    Differentiability of a function -- question on bounding

    Homework Statement I need to see if the function defined as##f(x,y) = \left\{ \begin{array}{lr} \frac{xy^2}{x^2 + y^2} & (x,y)\neq{}(0,0)\\ 0 & (x,y)=(0,0) \end{array} \right.## is differentiable at (0,0) Homework Equations [/B] A function is differentiable at a...
  19. R

    MHB Solving Limits: l'Hopital's Rule & Degree Rule

    Hi, I'm having some trouble with finding the limit for this question: I can use the l'hopital's rule which I tried.. I tried pi, 2pi, 0, inf, none seem to work so if I could have some help that would be appreciated! limx→0 \frac{cos5x-cos6x}{x^2} Would the degree rule apply here? It wouldn't...
  20. W

    I Manipulation of Integral limits

    I was looking over my lectures notes and became stumped when the lecturer began manipulating an integral using a dummy variable. I've attached the integral below as a JPEG. I can follow the substitution up until the very end but am having trouble understanding why he replaced u with a positive s...
  21. M

    Detecting Washing Machine Drum Out Of Balance Limits

    I am a noobie and I am not really sure this is the right forum to post to, so please excuse me if I am posting in the wrong place. I had considered General Physics and Classical Physics, but I hope I have made my best choice and I don't think it is acceptable to post in more than one forum...
  22. C

    Calculus Troubles: Finding Limits and Understanding 0/0

    Homework Statement Find the following limits, if they exist. Homework EquationsThe Attempt at a Solution I have just started calculus and am having trouble with 3 a). I get 0/0 after substitution so I factored but still get 0 in the denominator. Does this indicate that the limit does not...
  23. Mentz114

    Correlation limits for binary variates

    I've been looking at detector coincidences and tried to find what general limits apply to coincidences. I was surprised how simply the calculation works out. My question is whether it is correct and where can I find similar stuff ? Consider two binary sequences produced by random processes...
  24. zonde

    B Limits of no-communication theorem

    I would like to post a comment for offtopic conversation in another thread. This is the point of no-communication theorem that measuring one particle does not change anything measurable about the other particle. But conclusions of no-communication theorem are limited by it's assumptions (as for...
  25. Lucy788

    Solving Homework Problems: Evaluating Limits of f(X) as X->0

    hi I don't understand how to do one type of homework problem, here's an example of the type: If limit of f(X)/X = 1 as X ->0 evaluate the limit f(X) as X->0
  26. Q

    DiffEq, Binomial Expansion and limits

    Homework Statement Use algebra to show that U(x) = −√x − 1 and L(x) = −√x satisfy the ’funnel condition’ U(x) − L(x) → 0 as x → ∞ Homework Equations Funnel condition: The two fences come together asymptotically, i.e. U(x) − L(x) is small for large x. The Attempt at a Solution I think that...
  27. D

    Limits and Indeterminate Forms

    Is it true that every limit that takes on an indeterminate form can be evaluated? Is it proper to say that a limit problem has a solution if the limit does not exist?
  28. A

    Finding limits to a piecewise function (3 pieces)

    Homework Statement f(x)=-2 when x<1 =3 when x=1 =x-3 when x>1 find the limit at 1 from the left and right sides and at 1. Homework EquationsThe Attempt at a Solution limit for x when approaching 1 from the left is -2 limit for x when approaching 1 from the right is -2 -I'm not sure...
  29. S

    How do I prove that both are equivalent limits

    Homework Statement If k is a positive integer, then show that ##\lim_{x\to\infty} (1+\frac{k}{x})^x = \lim_{x\to 0} (1+kx)^\frac{1}{x}## Homework Equations L'Hopitals rule, Taylor's expansion The Attempt at a Solution How should I begin? Should I prove that both has the same limit, or is...
  30. Y

    Limits - Formula Validation/Verification

    Homework Statement The picture attached appeared in my powerpoint for my class. It's been a long time since I took calculus 1, but if I remember correctly this formula is wrong correct? I mean thinking about it limit k-> inf ( cos(theta)^k ) = 0 if theta is not a multiple of pi OR +/- 1 if...
  31. F

    Unusual Limit: Understanding the Discrepancy in the Integral of xe^-x

    This was just very basic, I have accepted it in just a heartbeat, but when I tried to chopped it and examined one by one, somethings fishy is happening, this just involved \int_{0}^{\infty}x e^{-x}dx=1. Well, when we do Integration by parts we will have let u = x du = dx dv = e^{-x}dx v =...
  32. kostoglotov

    Need help understanding how these limits were evaluated

    Homework Statement Hi, the problem is imply to show the following \lim_{n\rightarrow \infty} 10^n e^{-t} \sinh{10^{-n}t} = \lim_{n\rightarrow \infty} 10^n e^{-t} \sin{10^{-n}t} = te^{-t} How can I do this? Just a hint or a first step would be great, thanks :) Homework EquationsThe Attempt...
  33. Sirsh

    Understanding Limits - Spivak Calculus

    I have read Spivak's Calculus up to chapter 5, which is on Limits. Up until this point, the majority has been very straightforward and easy to understand. However, I am having trouble grasping the concept of limits in the style/method that Spivak describes them. Can anyone elaborate in a more...
  34. W

    Simplifying an expression using limits

    Homework Statement Given that ##T = \frac{4E(V-E)}{4E(V-E)+V^2\sinh^2 (ka)}## Simplify the expression for T when a is large but not infinite, and again for the case when a tends to zero and the potential tends to infinity, such that ##d = Va## is a real constant. k is a constant, as is E...
  35. J

    Implications of varying the definition of the derivative?

    I have been playing around with calculus for a while and I wondered what would it be like to make some changes to the definition of derivatives. I'd like to look at the original definition of derivatives in this way (everything is in lim Δx→0): F(x+Δx) - F(x) = F'(x) * Δx The Δx factor...
  36. Q

    What is the Limit of the Sequence b_n = n - sqrt(n^2 + 2n)?

    Homework Statement Consider the sequence given by b_{n} = n - \sqrt{n^{2} + 2n}. Taking (1/n) \rightarrow 0 as given, and using both the Algebraic Limit Theorem and the result in Exercise 2.3.1 (That if (x_n) \rightarrow 0 show that (\sqrt{x_n}) \rightarrow 0), show \lim b_{n} exists and find...
  37. Alpharup

    Question on ε in epsilon-delta definition of limits.

    I am using Spivak calculus. The reason why epsilon-delta definition works is for every ε>0, we can find some δ>0 for which definition of limit holds. Spivak asserts yhat if we can find a δ>0 for every ε>0, then we can find some δ1 if ε equals ε/2. How is this statement possible? Since ε>0, then...
  38. Alpharup

    Spivak problem on limits

    Consider the limit lim f(x)g(x) x→a Spivak has proved that this is equal to lim f(x) multlied by x→a lim g(x) x→a And also if lim g(x) = k and k≠0, x→a Then. lim 1/g(x) = 1/k x→a Now the...
  39. D

    Proper distance integral limits seem wrong

    I've seen in some lecture notes that the proper distance dp(t) can be written as ##\int_{t_e}^{t_0} c dt/a = \int_0^z c dz /H(z)## I can perform this integral ok using ##H =\dot a/a## and the fact that ##1 + z = 1/a(t_e)## but it requires associating the limits of the integration as te...
  40. ognik

    MHB Fourier Transform limits problem

    Find the Fourier Transform of $ e^{-a|t|}Cosbt $ I'd like to simplify this using $Cosbt = Re\left\{e^{ibt}\right\}$ $\therefore \hat{f}(\omega) = Re\left\{ \frac{1}{\sqrt{2\pi}}\int_{-\infty}^{\infty}e^{\left(-a+ib+iw\right)|t|} \,dt \right\} = Re\left\{ \frac{1}{\sqrt{2\pi}}...
  41. jfizzix

    Understanding Buoyancy: The Limits of Archimedes

    jfizzix submitted a new PF Insights post Understanding Buoyancy: The Limits of Archimedes Continue reading the Original PF Insights Post.
  42. Joseph Moore

    Weight limits when splitting the weight between two points

    I have a hammock that is rated to 400 lbs. I want to use two neodymium magnets to suspend the hammock from two steel posts. Do I need two magnets rated 400 lbs each?
  43. MAGNIBORO

    This Proof is right about sums and limits?

    hello, sorry for bad English, i have a question. if we consider the following equations and we take natural values note that tend 2 x-1=0 -----------------> x = 1 x^2-x-1=0 ----------------->...
  44. O

    MHB Exploring Limits and Infinite Subsets of $\Bbb{N}$

    İn a finite set, can we take limit to $\infty$ ? Also, can you give an example related to infinite subset of $\Bbb{N}$ ?
  45. shanepitts

    Exploring the Limits of Heat Engines with Ice Expansion

    Noting that ice expands by about 9%, why isn't it possible ot build a heat engine from this natural process?
  46. lordianed

    Prove that function tends to 0 everywhere in this interval

    Homework Statement From Spivak: Suppose that ##A_{n}## is, for each natural number ##n##, some finite set of numbers in ##[0,1]##, and that ##A_n## and ##A_m## have no members in common if ##m\neq n##. Define f as follows: ##f(x) = \frac{1}{n}##, if ##x \in A_n## ##f(x) = 0##, if ##x \notin...
  47. Steve Turchin

    Find Limits of a Function with Discontinuity | Homework Statement

    Homework Statement If ## lim_{x\to 0^+} f(x) = A ## and ## lim_{x\to 0^-} f(x) = B ##, find: (a) ## lim_{x\to 0^+} f(x^3-x) ## (b) ## lim_{x\to 0^-} f(x^3-x) ## Homework Equations None. Only the problem statement. The Attempt at a Solution (a) ## lim_{x\to 0^+} f(x^3-x) = lim_{x\to 0^+}...
  48. xwolfhunter

    A question about some concepts from solving limits

    Simple question, simple example. (x-1) \frac{(x-2)(x-1)}{x-2} So, it's quite clear that the contents of the two lines are not equal to one another. Since \frac{1}{1}\neq\frac{x-2}{x-2}, how is it the case that we can treat it as \frac{1}{1}=\frac{x-2}{x-2} when we're doing limits? Edit: Oh...
  49. wololo

    Multivariable continuity using limits

    Homework Statement Homework Equations lim(x,y)->(a,b)f(x,y) continuous at (a,b) if lim(x,y)->(a,b)f(x,y)=f(a,b) Squeeze theorem if lim a=lim c and lim a<= lim b <= lim c then lim b= lim c The Attempt at a Solution I proved that all the limits exist but somewhat the functions aren't all...
  50. thegirl

    Finding the limit of lim w-->wo ((exp(w)-exp(wo))/(w-wo))^-1

    Hi, I know that when you take this limit it is equal to e^-wo, but I was just wondering how you got there when taking the limit? lim w-->wo ((exp(w)-exp(wo))/(w-wo))^-1 = 1/e^wo w and wo are both two points within the same plane.
Back
Top